Please solve with explanation (high points)

Please Solve With Explanation (high Points)

Answers

Answer 1

Answer:

see the attachment photo!

Please Solve With Explanation (high Points)

Related Questions

FGH and HGJ form a linear pair. Find the measurement of the anglesif FGH=11x and HGJ+(6x-7)

Answers

The angles are 121 and 59 degrees, respectively

How to determine the angles?

The given parameters are:

FGH = 11x

HGJ = 6x - 7

Linear pair angles add up to 180.

So, we have:

11x + 6x - 7 = 180

Evaluate the like terms

17x = 187

Divide both sides by 17

x = 11

Substitute x = 11 in FGH = 11x and HGJ = 6x - 7

FGH = 11 * 11=121

HGJ = 6 * 11 - 7 = 59

Hence, the angles are 121 and 59 degrees, respectively


Read more about linear pairs at

https://brainly.com/question/2065148

#SPJ1

solve for y :
[tex]\longrightarrow \: \bold{3 + y = 18}[/tex]

ty! ~​

Answers

Answer:

y = 15

Step-by-step explanation:

Given :

3 + y = 18

This a simple algebraic equation.

Subtract 3 from both sides :

⇒ 3 + y - 3 = 18 - 3

y = 15

Answer:

y = 15

Step-by-step explanation:

Given equation:

3+y=18

To Find:

Value of y

Solution:

We can rewrite this equation as:

y+3 = 8

[This'll ain't change the answer]

Now we could solve on a easy way.

STEPS:

Transpose +3 to the RHS, make sure to change it's sign from “+” to “-”.

=> y = 18-3

Subtract the integers which's on the RHS:

=> y = 15

Hence,the value of y will be 15.

[tex] \rule{225pt}{2pt}[/tex]

a man is carrying load attached at the end of a stick load placed over his shoulder . show that the pressure at his shoulder is directly proportional to the distance between the shoulder and the load?​

Answers

According to the above information, it can be stated that the pressure on the man's shoulders is directly proportional to the distance between the shoulder and the load, since the greater the distance he will feel the more weight due to instability.

What is a directly proportional relationship?

A directly proportional relationship is a term that refers to the relationship between two variables in which if one of the variables increases, the other also increases.

How is the directly proportional relationship demonstrated in the situation presented?

According to the information, it can be inferred that the man will feel more weight on his shoulders if the load is more distant because this causes greater instability on his shoulders, so he will have to make a greater effort to move the load.

Learn more about proportionality in: https://brainly.com/question/8598338

#SPJ1

please help i dont know

Answers

Together they have 6 yards of fabric.

If f(x) = 2x + 3, what is f(–2)?

Answers

Substitute -2 for x

[tex]f( - 2) = 2( - 2) + 3 \\ y = - 4 + 3 \\ y = - 1[/tex]

Hope it helps

Please give brainliest

Answer:

-1

Step-by-step explanation:

Hi student! Let me help you out on this question.

_____________________

To find the value of f(-2), we need to stick in -2 for x.

[tex]\mathsf{f(-2)=2\cdot(-2)+3}[/tex]. Multiply first.

[tex]\mathrm{f(-2)=-4+3}[/tex]. Now simplify completely.

[tex]\mathsf{f(-2)=-1}[/tex]. Which is our final answer.

Hope that this helped you out! have a good day ahead.

Best Wishes!

[tex]\star\bigstar\underline{\underline{\overline{\overline{\textsf{Reach far. Aim high. Dream big.}}}}}\bigstar\star[/tex]

◆◈-Greetings!-◆◈

__________________

How is the product of 3 and –2 shown using integer tiles?
3 positive tiles and 2 negative tiles.
3 negative tiles and 2 positive tiles.
3 sets of 2 negative tiles.
3 sets of 2 positive tiles.

Answers

The answer is 3 sets of 2 negative tiles

The table gives information about the height of some trees draw a histogram for the information of some trees

Answers

2.3+2.2y uneed to pus the 2 on the 3 and answer is uten

Find the quartiles for these data values:

5,7,7,8,10,11,12,15,17
A. Q1=7, Q2=11, Q3=15
B. Q1=7, Q2=10, Q3=13.5
C. Q1=6, Q2=9, Q3=12
D. Q1=7.5, Q2=10.5, Q3=13.5

Answers

Answer: Choice B

Q1 = 7, Q2 = 10, Q3 = 13.5

=============================================================

Explanation:

Start with {5,7,7,8,10,11,12,15,17}

Notice how this data set is already sorted for us from smallest to largest.

Cross off the first and last items to get {7,7,8,10,11,12,15}

Repeat the last step to get this smaller set {7,8,10,11,12}

Repeat again: {8,10,11}

Repeat one more time: {10}

The 10 is at the very center, so it is the median aka the value of Q2.

-------------

An alternative way to get the median is to follow these steps:

There are n = 9 numbers in the original set before we crossed off any items. The middle number is in slot 5 because n/2 = 9/2 = 4.5 rounds up to 5. The value in the fifth slot is 10, so 10 is the median.

There are 4 items below the median and 4 items above it, giving n = 4+1+4 = 9 items total.

-------------

Next, break the data set into two smaller groups

L = lower set = every value below the median

L = {5, 7, 7, 8}

U = upper set = every value above the median

U = {11, 12, 15, 17}

The median itself is in neither set L nor set U.

The median of set L is (7+7)/2 = 7, so this is the value of Q1

The median of set U is (12+15)/2 = 13.5 which is the value of Q3

-------------

Summary:

Q1 = 7

Q2 = 10 (aka the median of the original set)

Q3 = 13.5

Answer:

B. Q1=7, Q2=10, Q3=13.5

Step-by-step explanation:

First Solution:

The first quartile of the data set is 7.

The first quartile (or lower quartile or 25th percentile) is the median of the bottom half of the numbers. So, to find the first quartile, we need to place the numbers in value order and find the bottom half.

5   7   7   8   10   11   12   15   17  

So, the bottom half is

5   7   7   8  

The median of these numbers is 7.

Second Solution:

The median of the data set is 10.

Quartile 2, also known as the median, is the middle number in a sorted list of numbers.   So, to find the median, we need to place the numbers in value order and find the middle number.

Ordering the data from least to greatest, we get:

5   7   7   8   10   11   12   15   17  

So, the median is 10 .

Third Solution:

The third quartile of the data set is 13.5.

The third quartile (or upper quartile or 75th percentile) is the median of the upper half of the numbers. So, to find the third quartile, we need to place the numbers in value order and find the upper half.

5   7   7   8   10  11   12   15   17  

So, the upper half is

11   12   15   17  

The median of these numbers is 13.5.

answer the problem please

Answers

Answer: Point C is (-2, -2), which is the third option.

Step-by-step explanation:

The midpoint formula is [tex]m=(\frac{x_{1}+x_{2} }{2},\frac{y_{1}+y_{2} }{2})[/tex]

All we have to do is plug in the numbers and solve the fractions from there.

[tex]m=(\frac{-10+6}{2},\frac{-6+2}{2} )[/tex]

[tex](\frac{-4}{2},\frac{-4}{2} )[/tex]

[tex]m=(-2, -2)[/tex]

I hope this helps!!

let a and b be roots of x² - 4x + 2 = 0. find the value of a/b² +b/a²​

Answers

Answer:

[tex]\dfrac a{b^2} + \dfrac b{a^2} = 10[/tex]

Step-by-step explanation:

[tex]\text{Given that, the roots are a,b and } ~ x^2 -4x+2 = 0\\\\\text{So,}\\\\a+b = -\dfrac{-4}1 = 4\\\\ab = \dfrac 21 = 2\\\\\text{Now,}\\\\~~~~~\dfrac a{b^2} + \dfrac b{a^2}\\\\\\=\dfrac{a^3 +b^3}{a^2b^2}\\\\\\=\dfrac{(a+b)^3 -3ab(a+b)}{(ab)^2}\\\\\\=\dfrac{4^3 -3(2)(4)}{2^2}\\\\\\=\dfrac{64-24}{4}\\\\\\=\dfrac{40}{4}\\\\\\=10[/tex]

Answer:

[tex]\dfrac{a}{b^2}+\dfrac{b}{a^2}=10[/tex]

Step-by-step explanation:

Given equation:   [tex]x^2-4x+2=0[/tex]

The roots of the given quadratic equation are the values of x when [tex]y=0[/tex].

To find the roots, use the quadratic formula:

[tex]x=\dfrac{-b \pm \sqrt{b^2-4ac} }{2a}\quad\textsf{when }\:ax^2+bx+c=0[/tex]

Therefore:

[tex]a=1, \quad b=-4, \quad c=2[/tex]

[tex]\begin{aligned}\implies x & =\dfrac{-(-4) \pm \sqrt{(-4)^2-4(1)(2)}}{2(1)}\\& =\dfrac{4 \pm \sqrt{8}}{2}\\& =\dfrac{4 \pm 2\sqrt{2}}{2}\\& =2 \pm \sqrt{2}\end{aligned}[/tex]

[tex]\textsf{Let }a=2+\sqrt{2}[/tex]

[tex]\textsf{Let }b=2-\sqrt{2}[/tex]

Therefore:

[tex]\begin{aligned}\implies \dfrac{a}{b^2}+\dfrac{b}{a^2} & = \dfrac{2+\sqrt{2}}{(2-\sqrt{2})^2}+\dfrac{2-\sqrt{2}}{(2+\sqrt{2})^2}\\\\& = \dfrac{2+\sqrt{2}}{6-4\sqrt{2}}+\dfrac{2-\sqrt{2}}{6+4\sqrt{2}}\\\\& = \dfrac{(2+\sqrt{2})(6+4\sqrt{2})+(2-\sqrt{2})(6-4\sqrt{2})}{(6-4\sqrt{2})(6+4\sqrt{2})}\\\\& = \dfrac{12+8\sqrt{2}+6\sqrt{2}+8+12-8\sqrt{2}-6\sqrt{2}+8}{36+24\sqrt{2}-24\sqrt{2}-32}\\\\& = \dfrac{40}{4}\\\\& = 10\end{aligned}[/tex]

Consider the linear system:

[tex]\overrightarrow y'=\begin{bmatrix}-6 & -4 \\12 & 8\end{bmatrix}\overrightarrow y[/tex]


a. Find the eigenvalues and eigenvectors for the coefficient matrix.

b. For each eigenpair in the previous part, form a solution of [tex]\overrightarrow y' = A\overrightarrow y[/tex]. Use [tex]t[/tex] as the independent variable in your answers.

c. Does the set of solutions you found form a fundamental set (i.e., linearly independent set) of solutions?

Answers

a. If A is the coefficient matrix, solve det(A - λI) = 0 for the eigenvalues λ :

[tex]\det\begin{bmatrix}-6-\lambda & -4 \\ 12 & 8-\lambda\end{bmatrix} = (-6-\lambda)(8-\lambda)+48 = 0 \implies \lambda(\lambda-2)=0[/tex]

[tex]\implies \lambda = 0, \lambda = 2[/tex]

Let v = [v₁, v₂]ᵀ be the eigenvector corresponding to λ. Solve Av = λv for v :

[tex]\lambda=0 \implies \begin{bmatrix}-6&-4\\12&8\end{bmatrix}\begin{bmatrix}v_1\\v_2\end{bmatrix} = \begin{bmatrix}0\\0\end{bmatrix} \implies 3v_1 + 2v_2 = 0[/tex]

If we pick v₂ = -3, then v₁ = 2, so [2, -3]ᵀ is the eigenvector for λ = 0.

[tex]\lambda = 2 \implies \begin{bmatrix}-8&-4\\12&6\end{bmatrix}\begin{bmatrix}v_1\\v_2\end{bmatrix} = \begin{bmatrix}0\\0\end{bmatrix} \implies 2v_1 + v_2 = 0[/tex]

Let v₁ = 1, so v₂ = -2.

b. λ = 0 and v = [2, -3]ᵀ contributes a constant solution,

[tex]\vec y_1 = e^{\lambda t} v = \begin{bmatrix}2\\-3\end{bmatrix}[/tex]

while λ = 2 and v = [1, -2]ᵀ contribute a solution of the form

[tex]\vec y_2 = e^{\lambda t} v = e^{2t} \begin{bmatrix}1\\-2\end{bmatrix}[/tex]

c. Yes; compute the Wronskian of the two fundamental solutions:

[tex]W(1, e^{2t}) = \det\begin{bmatrix}1 & e^{2t} \\ 0 & 2e^{2t}\end{bmatrix} = 2e^{2t} \neq 0[/tex]

The Wronskian is non-zero, so the solutions are independent.

√324 + 6 √64

what is the answer to this question.​

Answers

Answer:

66

Step-by-step explanation:

[tex]\sqrt{324} + 6 \sqrt{64}\\\\=\sqrt{18^2} +6 \sqrt{8^2}\\\\=18+6(8)\\\\=18+48\\\\=66[/tex]

Answer:

hmm

Step-by-step explanation:

not enough words to know u steal my points I steal yours kid

Which quadrilateral is not a parallelogram?
Rhombus
O Rectangle
Kite
Square

Answers

Answer:

Kite is not a parallelogram

Step-by-step explanation:

The sides aren't evenly lined up properly to be a parallelogram

ANSWER:

kite

STEP BY STEP EXPLANATION

A kite isnerally not a parallogram because it is a quadrilateral whose four sided can be grouped into two pairs of sideame length that arch other

The linear equation y = 25x describes how far from home Gary is as he drives from Montreal to Miami. Let x represent the number of hours and y represent the number of miles. How far from home is Gary in 12 hours? Graph the equation and tell whether it is linear.The linear equation y = 25x describes how far from home Gary is as he drives from Montreal to Miami. Let × represent the number of hours and y represent the number of miles. How far from home is Gary in 12 hours? Graph the equation and tell whether it is linear.

Answers

The distance that Greg is from home after 12 hours is given as follows:

300 miles.

How to calculate the numeric value of a function or of an expression?

To calculate the numeric value of a function or of an expression, we substitute each instance of any variable or unknown on the function by the value at which we want to find the numeric value of the function or of the expression presented in the context of a problem.

The function for this problem is given as follows:

y = 25x.

Hence the distance after 12 hours is given as follows:

y = 25 x 12

y = 300 miles.

Learn more about the numeric values of a function at brainly.com/question/28367050

#SPJ1

The sum of the three interior angles of a triangle is 180°. Suppose one angle is 68° and the remaining two angles are the same measure. Reason numerically to find the measures of the remaining angles.

Answers

Answer: 56 degree

Step-by-Step Explanation:

Let measure of unknown angle be ‘x’

One angle = 68 degree
Sum of all angles is 180 => 68 + 2x = 180

Therefore,
68 + 2x = 180
2x = 180 - 68
2x = 112
x = 112/2
=> x = 56

Therefore, the measure of the missing angles is 56 degrees each.

Answer:

56°

Step-by-step explanation:

Let ∠A and ∠B and ∠C.

be the three interior angles of the triangle.

∠A + ∠B + ∠C = 180

If ∠A = 68  and  ∠B = ∠C .

Then

∠B + ∠C = 180 - 68 = 112

Then

2∠B  = 112

Then

∠B = 112 ÷ 2 = 56°

Click on the pic there will be the question (very easy I’m just lazy lol)

Answers

Answer: ......... 14.85

Step-by-step explanation:

4.95 × 3

...

14.85

bros lazy lol

If an emitter current is changed by 4 mA, the collector current changes by 3.5 mA. The value of beta will be :​

Answers

Given -:

change in emitter current, ∆l(E) = 4 mA

change in collector current,∆I(C) = 3.5 mA

To find -:

value of B

Solution :-

∆I(E) = ∆I(C) + ∆l(B), here ∆l(B) is the change in base current.

4 = 3.5 + ∆l(B)

∆I(B) = 0.5 mA

Now, B = ∆l (C) ÷ ∆l (B)

B = 3.5 ÷ 0.5

B = 7.0

so, the value of A = 7.0

The point −20, 21 is on the terminal arm of an angle = in standard position. Find sin = and cos =.

Answers

The point −20, 21 is on the terminal arm of an angle = in standard position. Hence, sinФ = 21/29 and cos Ф = -20/29

What are trigonometric identities?

Trigonometric identities are the functions that include trigonometric functions such as sine, cosine, tangents, secant, and, cot.

Given Data

A (-20 , 21)

We have to find sin = ? and cos = ?

The point is in the fourth quadrant

We have the Opposite side and the Adjacent side

Calculate the hypotenuse

     c² = (-20)² + (21)²

       c² = 400 + 441

       c² = 841

       c = 29

Hence,

sinФ = 21/29

cos Ф = -20/29

Learn more about trigonometric;

https://brainly.com/question/21286835

#SPJ1

Determinar cuales de las siguientes frases son proposiciones
a) 3+2 = 0 c) ¡Hola!

b) x + 1 = 4 d) Yo estudio

Answers

The exercise is designed to test the students knowledge of prepositions. The correct answer thus is (Option D) Yo estudio (which translates) "I study".

What is the explanation for the answer above?

To understand the answer, you need to know what a preposition is. A preposition, in simple terms, is a word or group of words that comes before a noun.

The function of a preposition (much like an adjective) is to give clarity to the noun that it precedes.

Let us complete the Preposition phrase to give meaning to it:

"I study Mathematics". Where;

"Mathematics" is the noun;

"I Study" is the prepositional phrase. Hence the correction answer to the question above is Option D.

Learn more about prepositions at:

https://brainly.com/question/21537048

#SPJ1


Which choice shows 19 + 32 + 11 rewritten correctly using the commutative property and then simplified correctly?
19+43 = 62
19+11+32= 30+32 = 62
11+9+10+32 20+32 = 52
19+11+32 = 30 +43 = 73

Answers

The choice shows 19 + 32 + 11 rewritten correctly using the commutative property and then simplified correctly is option B; 19+11+32= 30+32 = 62.

What is the commutative property of addition?

The commutative property of addition says that it doesn't matter how we add two numbers, the result of the addition would be same.

For two numbers x and y, we have:

x + y = y + x

WE have given

19 + 32 + 11

The sum would be 62.

We know that by commutative property of addition,

For two numbers x and y,

x + y = y + x

Thus, if we take 3 numbers as a,b and c, then:

c + a + b = c + b + a

Similarly,

19 + 32 + 11 = 19+11+32

= 30+32

= 62

Therefore, the option B  is the correct answer.

Learn more about properties of addition here:

https://brainly.com/question/1709448

#SPJ1

In function notation, f(x)is used instead of the letter ___ to represent the __________ variable.

Answers

In function notation, f(x) is used instead of the letter y to represent the output variable.

How to complete the blanks?

A function is represented as:

f(x)

The above means that:

The function of x

As a general rule, the function can be rewritten using the letter y (i.e. the output variable)

Hence, the words that complete the blanks are y and output

Read more about functions at:

https://brainly.com/question/4025726

#SPJ1

[tex]\frac{1}{2} (5x - 9 ) = 2 (\frac{1}{3} + 6 )[/tex]

Answers

Answer: 103/15

Step-by-step explanation:

We can simplify the right-hand side to be [tex]2 \left(\frac{1}{3}+6 \right)=2 \left(\frac{19}{3} \right)=\frac{38}{3}[/tex].

This means we need to solve:

[tex]\frac{1}{2}(5x-9)=\frac{38}{3}\\5x-9=\frac{76}{3}\\5x=\frac{103}{3}\\x=\boxed{\frac{103}{15}}[/tex]

HELP NEEDED ASAP
answer this question for me, image below

Answers

Answer: Heyaa! ~

Your Answer Is... 13

Step-by-step explanation:

385 divided by 28 = 13. 75Reduce the expression, if possible, by cancelling the common factors.

Hopefully this helps you! ~

Solve 24x ⋅ 321-x = 8x+2

Answers

Answer:

[tex]x=2/7695[/tex]

≡ Small number, yes, but the steps explain why x is equal to this.

Step-by-step explanation:

simplify

[tex]24x\cdot 321-x=8x+2\\\left(24\cdot 321\right)x-x=8x+2\\7703x=8x+2[/tex]

group

[tex]7703x=8x+2\\7703x-8x=8~x+2-8x\\7695x=8x+2-8x\\7695x=8x-8x+2\\7695x=2[/tex]

isolate

[tex]7695x=2\\\frac{7695x}{7695}=\frac{2}{7695}\\x=\frac{2}{7695}[/tex]

The fraction is in its simplest form, so that is the final answer.

Findthe domain of the function f(x) = √19-x​

Answers

Answer:

(-∞,19)

Definition: The domain of a function is the set of input or argument values for which the function is real and defined

Answer:

(-∞, 19]

Step-by-step explanation:

The domain of this function is only true for all real values of f(x).

That means the lower limit of the function is -∞ as the values are positive, and the upper limit is 19, because that is the greatest value it can become before becoming negative.

The domain is : (-∞, 19]

I don’t quite understand this problem could someone help me please

Answers

Answer:

[tex]\frac{7\sqrt{65}}{65}[/tex]

Step-by-step explanation:

Cosine is the ratio of the side adjacent to the angle and the right triangle hypotenuse.

[tex]cos[/tex] B = [tex]\frac{7}{\sqrt{65}}[/tex] = [tex]\frac{7\sqrt{65}}{65}[/tex]

6 children were jumping rope.
5 children joined them.
2 children left because the line was too long.
How many children were jumping rope then

Answers

Answer:

9 children were jumping then.

Step-by-step explanation:

6+5 equals to 11 - 2 of the children left would equal 9 children left.

What is the output of this program?

numA = 2
numB = 3
if numA == 2 or numB == 2:
print("yes")
elif numA == 2 and numB == 3:
print("no")
Output:




PLEASE HELP ME PLEASE

Answers

Since the compiler reads top down, it should print “yesno” because they are both true

with full explanation from the internet like before
1/(x-5)+3/(x+2)=4

Answers

Solution :

[tex]x = \frac{4 + \sqrt{43} }{2} .x = 4 + \frac{4 - \sqrt{43} }{2} [/tex]

Step-by-step explanation:

[tex] \frac{1}{(x - 5)} + \frac{3}{(x - 2)} - 4[/tex]

1. Multiply by LCM

[tex]x = 2 + 3( x - 5) = 4 - (x - 5)(x + 2)[/tex]

2. Solve[tex]x = 2 + 3(x - 5) = 4 (x - 5)(x + 2)[/tex]

[tex]x = \frac{4 + \sqrt{43} }{2} .x = 4 + \frac{4 - \sqrt{43} }{2} [/tex]3.Verify Solutions

Find undefined (singularity) points : x=5,x=–2

[tex]x = \frac{4 + \sqrt{43} }{2} .x = 4 + \frac{4 - \sqrt{43} }{2} [/tex]

[tex]\\ \rm\Rrightarrow \dfrac{1}{x-5}+\dfrac{3}{x+2}=4[/tex]

[tex]\\ \rm\Rrightarrow \dfrac{x+2+3x-15}{(x-5)(x+2)}=4[/tex]

[tex]\\ \rm\Rrightarrow 4x-13=4(x-5)(x+2)[/tex]

[tex]\\ \rm\Rrightarrow 4x-13=4(x(x+2)-5(x+2))[/tex]

[tex]\\ \rm\Rrightarrow 4x-13=4(x^2+2x-5x-10)[/tex]

[tex]\\ \rm\Rrightarrow 4x-13=4(x^2-3x-10)[/tex]

[tex]\\ \rm\Rrightarrow 4x-13=4x^2-12x-40[/tex]

[tex]\\ \rm\Rrightarrow 4x^2-16x-53=0[/tex]

On solving we get

[tex]\\ \rm\Rrightarrow x=2\pm\dfrac{69}{2}[/tex]

x²-12x+36
A. (x-6)(x+6)
B. (x+9)(x-4)
C. (x+6)(x+6)
D. (x-6)(x-6)
SUBMIT

Answers

Answer:

x²-12x+36

x²-6x-6x+36

x(x-6)-6(x-6)

(x-6)(x-6)

=(x-6)²

Other Questions
Michael and Zach are hiking in the mountains. They left Michaels car in the parking lot and walked northwest for 12.4 km to a campsite. Then they turned due south and walked another 7.0 km to a glacier lake. The weather was taking a turn for the worse, so they decided to plot a course directly back to the parking lot. Zach remembered, from the map in the parking lot, that the angle between the path to the campsite and the path to the glacier lake measures about 30. What compass direction should they follow to return directly to the parking lot? Question 1 of 5Which two substances are products of the chemical reactions of cellularrespiration?A. OxygenB. Carbon dioxideC. SugarD. WaterSUBMIT Question 4 1 pts in 2013, the number of rabbits in a local park is 120, but this number is decreasing each year by a constant percent. the number of rabbits in the park t years after 2013 can be found using the given formula. about how many rabbits will be left in the park after 10 years? r=120(0.99) 1,188 247 1 108 ASAP!!! Answer this question please! I'll give the brainilest! ^^Explain the process of Soil formation, including the different components of healthy soil. Viruses attack and destroy which of the following(A) cells(B) mutations(C) vaccines(D) pasteurization Please click on image to see questions. Greatly appreciated. A market economy is driven by the economic principle of. Which words could replace the subordinating conjunction in the sentence while maintaining its original structure? choose two answers. and because but since so What is the area of a polygon with vertices of (-2,-4), (4,-4), (4, 4), and (-5, 4)?216 square units30 square units120 square units60 square units f(x) =2x^2+4x-6. g(x)=4x^3-6x^2+3 find (f+g)(x) Name the organs which form a part of our respiratory system. At what rate per cent per annum will rs 6000 amount to rs 6615 in two years when interest is compounded annually? the process of transferring files from computer to internet is called.... Put the words in alphabetical order. Gold (Au) should have an electron notation that ends in 6s24f145d9. To become more stable, the electrons in reality rearrange themselves. How do they do it If the perimeter of this triangle is 15 centimeters, what is the value of n? Find sin A for the triangle below. Give the exact value as an expression and an approximation to the nearest ten-thousandth. Note: The triangle is not drawn to scale. how did radical islamic fundamentalism contribute to the growth of al-qaeda Evaluate the expression when x = 6.x +8x-3 Which is an adaptation plants have developed because of gravity?vines curl around treesroots grow into the groundleaves curlingplants lean